Difference between revisions of "2002 AMC 12A Problems/Problem 17"

(Solution)
m (Solution)
 
Line 22: Line 22:
  
 
Thus the answer is <math>207\implies \boxed{\mathrm{(B)}}</math>.
 
Thus the answer is <math>207\implies \boxed{\mathrm{(B)}}</math>.
 +
 +
== Video Solution ==
 +
 +
https://www.youtube.com/watch?v=V6z7GiitBUM
  
 
== See Also ==
 
== See Also ==

Latest revision as of 15:01, 8 September 2022

Problem

Several sets of prime numbers, such as $\{7,83,421,659\}$ use each of the nine nonzero digits exactly once. What is the smallest possible sum such a set of primes could have?

$\text{(A) }193 \qquad \text{(B) }207 \qquad \text{(C) }225 \qquad \text{(D) }252 \qquad \text{(E) }447$

Solution

Neither of the digits $4$, $6$, and $8$ can be a units digit of a prime. Therefore the sum of the set is at least $40 + 60 + 80 + 1 + 2 + 3 + 5 + 7 + 9 = 207$.

We can indeed create a set of primes with this sum, for example the following sets work: $\{ 41, 67, 89, 2, 3, 5 \}$ or $\{ 43, 61, 89, 2, 5, 7 \}$.

Thus the answer is $207\implies \boxed{\mathrm{(B)}}$.

Video Solution

https://www.youtube.com/watch?v=V6z7GiitBUM

See Also

2002 AMC 12A (ProblemsAnswer KeyResources)
Preceded by
Problem 16
Followed by
Problem 18
1 2 3 4 5 6 7 8 9 10 11 12 13 14 15 16 17 18 19 20 21 22 23 24 25
All AMC 12 Problems and Solutions

The problems on this page are copyrighted by the Mathematical Association of America's American Mathematics Competitions. AMC logo.png